-
Notifications
You must be signed in to change notification settings - Fork 13
Commit
This commit does not belong to any branch on this repository, and may belong to a fork outside of the repository.
- Loading branch information
Showing
15 changed files
with
1,789 additions
and
43 deletions.
There are no files selected for viewing
This file contains bidirectional Unicode text that may be interpreted or compiled differently than what appears below. To review, open the file in an editor that reveals hidden Unicode characters.
Learn more about bidirectional Unicode characters
Original file line number | Diff line number | Diff line change |
---|---|---|
@@ -0,0 +1,5 @@ | ||
\section*{Предупреждение} | ||
|
||
Единственный, кто несёт ответственность материал в этом конспекте --- его автор. Он не может гарантировать верность ни утверждений, ни приведённых доказательств, однако старается исправлять все поступающие замечания по мере сил. | ||
|
||
Если вы увидели несостыковки либо ошибки, то смело сообщайте (имя автора на титульной странице кликабельно). |
189 changes: 189 additions & 0 deletions
189
Lectures/6_Semester/Random_Processes/2024_Bogachev/lectures/10lecture.tex
This file contains bidirectional Unicode text that may be interpreted or compiled differently than what appears below. To review, open the file in an editor that reveals hidden Unicode characters.
Learn more about bidirectional Unicode characters
Original file line number | Diff line number | Diff line change |
---|---|---|
@@ -0,0 +1,189 @@ | ||
\section{Ветвящиеся процессы Гальтона-Ватсона} | ||
|
||
\begin{definition} | ||
\textit{Процессом Гальтона-Ватсона} называется однородная цепь Маркова $Z$ с фазовым пространством $E = \N_0$, начальным распределением $\pi_0$ и переходными вероятностями следующего вида: | ||
\[ | ||
p_{k, j} = P(Z_{n + 1} = j | Z_n = k) = p_j^{*k}, k > 0 | ||
\] | ||
при этом $p_{0, 0} = 1$, $p_{0, j} = 0$ и | ||
\[ | ||
p_j^{*k} = \sum_{j_1 \plusdots j_k = j} p_{j_1} \cdot \ldots \cdot p_{j_k} | ||
\] | ||
где $(p_0, p_1, \ldots)$ --- вероятностное распределение на $\N_0$. | ||
\end{definition} | ||
|
||
\begin{note} | ||
$\pi_0$, вообще говоря, может быть любым, но далее мы предполагаем детерминрованный вариант $\pi_0(1) = 1$. | ||
\end{note} | ||
|
||
\begin{note} | ||
Процесс Гальтона-Ватсона можно записать и другим образом. Пусть $\{\xi_k\}_{k = 1}^\infty$ --- независимые случайные величины с общим распределением: | ||
\[ | ||
P(\xi_k = m) = p_m | ||
\] | ||
Тогда $p_j^{*k} = P(\xi_1 \plusdots \xi_k = j)$. | ||
\end{note} | ||
|
||
\begin{note} | ||
Интерпретация рассматриваемого процесса Гальтона-Ватсона такова: наблюдается процесс смены поколений частиц. Частица живёт 1 единицу времени, и в начале была ровно 1 частица. Каждая частица может породить произвольное число потомков согласно вероятностному распределению $(p_0, p_1, \ldots)$. На моменте времени 2 уже, естественно, будет не одна частица, но все они могут породить потомков по этому же распределению, и так далее. | ||
|
||
Число частиц в $n$-м поколении есть $Z_n$. Если $\xi_k^{(n)}$ --- случайная величина, отвечающая количеству потомков от $k$-й частицы в $(n - 1)$-м поколении, то | ||
\[ | ||
Z_n = \sum_{k = 1}^{Z_{n - 1}} \xi_k^{(n)}, n \ge 1;\ Z_0 = 1 | ||
\] | ||
\end{note} | ||
|
||
\begin{problem} | ||
Найти вероятность вырождения процесса Гальтона-Ватсона, то есть вероятность того, что все частицы умрут: | ||
\[ | ||
q = P(\exists n \in \N \colon Z_n = 0) | ||
\] | ||
Также должно быть понятно, что $q = \lim_{n \to \infty} q_n$, где $q_n := P(Z_n = 0)$. | ||
\end{problem} | ||
|
||
\begin{note} | ||
Отсюда и до конца параграфа, если не сказано явно другого, любая случайная величина рассматривается со значениями в $\N_0$. | ||
\end{note} | ||
|
||
\begin{definition} \textcolor{red}{(не по лектору)} | ||
\textit{Производящей функцией случайной величины $\xi$} называется производящая функция, у которой коэффициенты при степени $z^k$ соответствуют $P(\xi = k)$. | ||
\end{definition} | ||
|
||
\begin{proposition} | ||
Пусть $\xi$ --- случайная величина. Тогда её производящая функция может быть записана как | ||
\[ | ||
\forall z \in \Cm, |z| \le 1\ \ \phi_\xi(z) = \E z^\xi = \sum_{k \ge 0} P(\xi = k)z^k | ||
\] | ||
\end{proposition} | ||
|
||
\begin{proof} | ||
Тривиально | ||
\end{proof} | ||
|
||
\begin{proposition} | ||
Пусть $\xi$ --- случайная величина. Тогда производящая функция $\phi_\xi(z) = \E z^\xi$ определена в замкнутом единичном круге, задаёт аналитическую функцию на внутренности и непрерывную на замыкании, причём $\phi_\xi(1) = 1$ и $\phi'_\xi(1) = \E\xi$ | ||
\end{proposition} | ||
|
||
\begin{note} | ||
Если в производящей функции положить $z = e^{it}$, то получится преобразование Фурье. | ||
\end{note} | ||
|
||
\begin{lemma} (рекуррентность харфункции процесса Гальтона-Ватсона) | ||
Имеет место формула: | ||
\[ | ||
\forall z \in \Cm, |z| \le 1\ \ \phi_{Z_n}(z) = \phi_{Z_{n - 1}}(\phi_\xi(z)) | ||
\] | ||
\end{lemma} | ||
|
||
\begin{proof} | ||
Заметим, что $|\phi_\xi(z)| \le 1$ при $|z| \le 1$. Найдём вид для $\E(z^{Z_n} | Z_{n - 1})$: | ||
\begin{multline*} | ||
\E(z^{Z_n} | Z_{n - 1} = m) = \E\ps{z^{\sum_{k = 1}^{Z_{n - 1}} \xi_k^{(n)}} | Z_{n - 1} = m} = \E\ps{z^{\sum_{k = 1}^m \xi_k^{(n)}} | Z_{n - 1} = m} = [1] = | ||
\\ | ||
\E\ps{z^{\sum_{k = 1}^m \xi_k^{(n)}}} = \E\ps{\prod_{k = 1}^m z^{\xi_k^{(n)}}} = \prod_{k = 1}^m \E(z^{\xi_k^{(n)}}) = \phi_\xi(z)^m | ||
\end{multline*} | ||
Пояснение 1: $Z_{n - 1} = \sum_{k = 1}^{Z_{n - 2}} \xi_k^{(n - 1)}$ по определению, то есть $Z_{n - 1}$ зависит только от $\xi_k^{(l)}$, где $l \le n - 1$. Значит, $Z_{n - 1}$ независимо со всеми $\xi_k^{(n)}$. Итого: | ||
\[ | ||
\E(z^{Z_n} | Z_{n - 1}) = \phi_\xi(z)^{Z_{n - 1}} | ||
\] | ||
Применяя формулу полной вероятности в рамках УМО, получим | ||
\[ | ||
\phi_{Z_n}(z) = \E z^{Z_n} = \E(\E(z^{Z_n} | Z_{n - 1})) = \E(\phi_\xi(z)^{Z_{n - 1}}) = \phi_{Z_{n - 1}}(\phi_\xi(z)) | ||
\] | ||
\end{proof} | ||
|
||
\begin{corollary} | ||
Для $\phi_{Z_n}(z)$ имеет место формула | ||
\[ | ||
\phi_{Z_n}(z) = \underbrace{\phi_\xi(\cdots \phi_\xi(\phi_\xi}_{n}(z)) \cdots) | ||
\] | ||
Отсюда же $\phi_{Z_n}(z) = \phi_\xi(\phi_{Z_{n - 1}}(z))$ | ||
\end{corollary} | ||
|
||
\begin{proof} | ||
Проведём индукцию по $n$: | ||
\begin{itemize} | ||
\item База $n = 0$: $\phi_{Z_0}(z) = \E z^1 = z$ | ||
|
||
\item Переход $n > 0$: | ||
\[ | ||
\phi_{Z_n}(z) = [\text{лемма}] = \phi_{Z_{n - 1}}(\phi_\xi(z)) = \underbrace{\phi_\xi(\cdots \phi_\xi}_{n - 1}(\phi_\xi(z)) \cdots) | ||
\] | ||
\end{itemize} | ||
\end{proof} | ||
|
||
\begin{lemma} | ||
Вероятность вырождения $q$ является неподвижной точкой $\phi_\xi$: | ||
\[ | ||
q = \phi_\xi(q) | ||
\] | ||
\end{lemma} | ||
|
||
\begin{proof} | ||
В силу рекуррентных формул для производящей функции $Z_n$: | ||
\[ | ||
q_n := P(Z_n = 0) = \phi_{Z_n}(0) = \phi_\xi(\phi_{Z_{n - 1}}(0)) = \phi_\xi(q_{n - 1}) | ||
\] | ||
Сделаем предельный переход с обеих сторон: | ||
\[ | ||
\lim_{n \to \infty} q_n = q = \lim_{n \to \infty} \phi_\xi(q_{n - 1}) = \phi_\xi(\lim_{n \to \infty} q_{n - 1}) = \phi_\xi(q) | ||
\] | ||
\end{proof} | ||
|
||
\begin{note} | ||
Из вышесказанного совершенно не обязательно, что $q = 1$, ведь у $\phi_\xi$ может быть много неподвижных точек. | ||
\end{note} | ||
|
||
\begin{theorem} | ||
Пусть $P(\xi = 1) < 1$. Тогда | ||
\begin{itemize} | ||
\item Если $\E\xi \le 1$, то уравнение $z = \phi_\xi(z)$ имеет единственное решение на $[0; 1]$. (Следовательно, $q = 1$) | ||
|
||
\item Если $\centernot\exists \E\xi$ или $\E\xi > 1$, то уравнение $z = \phi_\xi(z)$ имеет единственное решение на $\lsi{0; 1}$. (Причём $q = z_0 \in \lsi{0; 1}$) | ||
\end{itemize} | ||
\end{theorem} | ||
|
||
\begin{note} | ||
Таким образом, $q$ всегда равно наименьшему корню уравнения $z = \phi_\xi(z)$ на $[0; 1]$. | ||
\end{note} | ||
|
||
\begin{proof} | ||
Для начала отметим, что функция $\phi_\xi$ выпукла вниз на $\lsi{0; 1}$: | ||
\[ | ||
\forall x \in \lsi{0; 1}\ \ \phi'_\xi(x) = \sum_{k \ge 1} P(\xi = k)kx^{k - 1} \ge 0;\ \ \phi''_\xi(x) = \sum_{k \ge 2} P(\xi = k)k(k - 1)x^{k - 2} \ge 0 | ||
\] | ||
Причём $\phi_\xi(0) = P(\xi = 0) \ge 0$, $\phi_\xi(1) = 1$. Произведём большой разбор случаев: | ||
\begin{itemize} | ||
\item $P(\xi = 0) = 1$. Тогда $\phi_\xi = 1$, $\E\xi = 0$ тривиальным образом | ||
|
||
\item $P(\xi = 0) < 1 \wedge \forall x \in \lsi{0; 1}\ \phi''_\xi(x) = 0$. Тогда, можно решить дифференциальное уравнение и найти $\phi_\xi(x) = P(\xi = 0) + xP(\xi = 1)$. Отсюда $\E \xi \le 1$ и $x = 1$ --- единственный корень уравнения $x = \phi_\xi(x)$ | ||
|
||
\item $P(\xi = 0) < 1 \wedge \phi''_\xi|_{\lsi{0; 1}} \neq 0$. Из формы $\phi''_\xi$ в виде ряда видно, что \\ $\forall x \in (0; 1)\ \phi''_\xi(x) > 0$, поэтому кроме корня 1 может быть только ещё один корень $x_0 \in \lsi{0; 1}$. | ||
\begin{itemize} | ||
\item $\phi'_\xi(1) = \E\xi \le 1$. Тогда, второго корня нет, ибо $\phi_\xi(z) \ge z$ при $z \in [0; 1]$ из-за выпуклости вниз. | ||
|
||
\item $\E\xi > 1$ либо $\E\xi = +\infty$. Тогда, есть ровно один корень $x_0 \in \lsi{0; 1}$. Покажем, что $q = x_0$ в таком случае. Для этого достаточно проверить, что $q_n \le x_0$. Сделаем это по индукции: | ||
\begin{itemize} | ||
\item База $n = 0$: $q_0 = \phi_\xi(0) \le \phi_\xi(x_0) = x_0$. | ||
|
||
\item Переход $n > 0$: $\phi_\xi(q_{n - 1}) = q_n \ge q_{n - 1}$, поэтому $\phi_\xi(q_n) - q_n \ge 0$, а отсюда сразу $q_n \le z_0$ в силу известных свойств $\phi_\xi$. | ||
\end{itemize} | ||
\end{itemize} | ||
\end{itemize} | ||
\end{proof} | ||
|
||
\begin{example} | ||
Рассмотрим бинарное деление частиц: с вероятностью $p$ частица производит 2х потомков, либо с вероятностью $1 - p$ просто умирает. Тогда, вероятность вырождения --- это корень уравнения | ||
\[ | ||
x = (1 - p) + px^2 | ||
\] | ||
Отсюда $q = (1 - p) / p$ при $p > 1 / 2$ и $q = 1$ иначе. | ||
\end{example} | ||
|
||
\begin{example} | ||
В случае деления частиц с геометрическим распределением ($P(\xi = k) = (1 - p)p^k$, $p \in (0; 1)$), получится уравнение | ||
\[ | ||
x = \frac{1 - p}{1 - xp} | ||
\] | ||
В результате $q$ принимает те же значения, что и в предыдущем примере. Величину $p$ можно однозначно определить по $\E\xi = p / (1 - p)$, поэтому если $\E\xi = 6.5$, то уже $q = 5 / 6 > 0.8$ (огромная вероятность вырождения). | ||
\end{example} |
113 changes: 113 additions & 0 deletions
113
Lectures/6_Semester/Random_Processes/2024_Bogachev/lectures/11lecture.tex
This file contains bidirectional Unicode text that may be interpreted or compiled differently than what appears below. To review, open the file in an editor that reveals hidden Unicode characters.
Learn more about bidirectional Unicode characters
Original file line number | Diff line number | Diff line change |
---|---|---|
@@ -0,0 +1,113 @@ | ||
\section{Модель страхования Крамера-Лундберга} | ||
|
||
\begin{problem} | ||
Предположим, что мы --- страховая компания. Помимо желания заработать, нам также важно понимать, можем ли мы разориться, и если да, то когда. Пусть $T = \R_+$, $X_t$ --- размер капитала в момент времени $t \in T$, $c > 0$ --- скорость поступления страховых взносов, $\eta_0 > 0$ --- начальный капитал, а $\eta_k$ --- сумма выплат по $k$-му страховому случаю и $N_t$ означает число таких случае до момента $t$ включительно. Тогда, капитал можно описать следующей формулой: | ||
\[ | ||
X_t = \eta_0 + ct - \sum_{k = 1}^{N_t} \eta_k | ||
\] | ||
Вполне логично ожидать, что $(N_t)_{t \in T}$ является пуассоновским процессом интенсивности $\lambda > 0$, $\{\eta_k\}_{k = 1}^\infty$ --- независимые одинаково распределённые неотрицательные величины, причём эта последовательность также независима с $N$. Момент разорения $\tau \colon \Omega \to T \cup \{+\infty\}$ можно задать так: | ||
\[ | ||
\tau(\omega) = \inf \{t \in T \colon X_t < 0\} | ||
\] | ||
Соответствено, вся задача свелась к изучению $P(\tau < +\infty)$. Также мы потребуем, что $a = \E\eta_1 > 0$, $c - \lambda a > 0$ и $\psi(v) = \E e^{v\eta_1} < \infty$ для всех $v > 0$. | ||
\end{problem} | ||
|
||
\begin{proposition} | ||
У процесса $X$ независимые приращения и непрерывные справа траектории. | ||
\end{proposition} | ||
|
||
\begin{proof}~ | ||
\begin{itemize} | ||
\item (Независимость приращений) Рассмотрим $0 \le t_1 < t_2 < t_3 < t_4$. Найдём вид $X_{t_2} - X_{t_1}$: | ||
\[ | ||
X_{t_2} - X_{t_1} = c(t_2 - t_1) - \sum_{k = 1}^{N_{t_2}} \eta_k + \sum_{k = 1}^{N_{t_1}} \eta_k | ||
\] | ||
В силу того, что приращения пуассоновского процесса принимают неотрицательные значения, $N_{t_1} \le N_{t_2}$. Тогда | ||
\[ | ||
X_{t_2} - X_{t_1} = c(t_2 - t_1) - \sum_{k = N_{t_1} + 1}^{N_{t_2}} \eta_k | ||
\] | ||
Отсюда видно, что $X_{t_2} - X_{t_1} = \phi(\eta_{N_{t_1} + 1}, \ldots \eta_{N_{t_2}})$. Этот набор не пересекается с набором для $X_{t_4} - X_{t_3}$, поэтому приращения обязаны быть независимыми. \textcolor{red}{Это не совсем правда, может быть ситуация $N_{t_2} = N_{t_3}$, да и даже так, наборы случайных величин не фиксированы} | ||
|
||
\item (Непрерывность справа траекторий) Зафиксируем $\omega \in \Omega$. Тогда | ||
\[ | ||
X(t, \omega) - X(t_0, \omega) = c(t - t_0) - \sum_{k = N(t_0, \omega)}^{N(t, \omega)}\eta_k(\omega) | ||
\] | ||
Часть $c(t - t_0)$ тривиально будет стремится к нулю при $t \to t_0+$. Так как слагаемые под знаком суммы не зависят от $t$, вопрос становится лишь в том, почему $N(t, \omega) - N(t_0, \omega)$ будет стремится к нулю при $t \to t_0+$ для почти всех $\omega \in \Omega$, а это известный факт: у пуассоновского процесса $P$-почти наверное траектории непрерывны справа. | ||
\end{itemize} | ||
\end{proof} | ||
|
||
\begin{lemma} (без доказательства) | ||
Пусть $s < t$. Тогда | ||
\[ | ||
\E\exp(-v(X_t - X_s)) = \exp((t - s)g(v)),\ g(v) = \lambda(\psi(v) - 1) - vc | ||
\] | ||
\end{lemma} | ||
|
||
\begin{proof} | ||
Подставим процесс $X$: | ||
\[ | ||
X_t - X_s = c(t - s) - \sum_{k = N_s + 1}^{N_t} \eta_k | ||
\] | ||
Тогда | ||
\[ | ||
\E\exp(-v(X_t - X_s)) = e^{-vc(t - s)}\E\exp\ps{v\sum_{k = N_s + 1}^{N_t} \eta_k} = e^{-vc(t - s)}\E\ps{\prod_{k = N_s + 1}^{N_t} e^{v\eta_k}} | ||
\] | ||
Последнее математическое ожидание можно посчитать через разбиение $\Omega$ на подмножества $B_{m_s, m_t} = \{N_s = m_s \le m_t = N_t\}$. Тогда | ||
\begin{multline*} | ||
\E\ps{\prod_{k = N_s + 1}^{N_t} e^{v\eta_k} \cdot \chi_{B_{m_s, m_t}}} = \E\ps{\prod_{k = m_s + 1}^{m_t} e^{v\eta_k}\chi_{B_{m_s, m_t}}} = \prod_{k = m_s + 1}^{m_t} \E(e^{v\eta_k}\chi_{B_{m_s, m_t}}) = | ||
\\ | ||
[\text{$\eta_k$ одинаково распр.}] = \prod_{k = m_s + 1}^{m_t} \E(e^{v\eta_1}\chi_{B_{m_s, m_t}}) = \E(e^{v\eta_1}\chi_{B_{m_s, m_t}})^{m_t - m_s} | ||
\end{multline*} | ||
Отсюда | ||
\[ | ||
\E\ps{\prod_{k = N_s + 1}^{N_t} e^{v\eta_k}} = \sum_{m_s \le m_t} \E(e^{v\eta_1}\chi_{B_{m_s, m_t}})^{m_t - m_s} | ||
\] | ||
\textcolor{red}{Допридумать} | ||
\end{proof} | ||
|
||
\begin{proposition} | ||
Случайный процесс $M_t = \exp(-vX_t - tg(v))$ является мартингалом относительно фильтрации $(\F_t)_{t \in T}$, порождённой $X$. | ||
\end{proposition} | ||
|
||
\begin{proof} | ||
Согласованность очевидна. Свойство сужения проверяется классическим образом, через выделение приращения: | ||
\begin{multline*} | ||
\E(M_t | \F_s) = \E\big(\exp(-v(X_t - X_s) - vX_s -tg(v)) | \F_s\big) = | ||
\\ | ||
\exp(-vX_s - tg(v))\E(\exp(-v(X_t - X_s)) | \F_s) = | ||
\\ | ||
[\text{приращения незав.}] = \exp(-vX_s - tg(v))\E\exp(-v(X_t - X_s)) = | ||
\\ | ||
[\text{лемма}] = \exp(-vX_s - tg(v) + (t - s)g(v)) = \exp(-vX_s - sg(v)) = M_s | ||
\end{multline*} | ||
\end{proof} | ||
|
||
\begin{solution} | ||
Итак, мы бы снова хотели пользоваться теоремой об остановке, но для $\tau$ ничего не знаем. Зато для всякого $t > 0$ величина $\tau_t = \min\{t, \tau\}$ является ограниченным моментом остановки. Отсюда | ||
\begin{multline*} | ||
e^{-v\eta_0} = M_0 = \E M_0 = [\text{т. об остановке}] = \E M_{\tau_t} \ge \E M_{\tau_t}\chi_{\tau \le t} = \E M_\tau\chi_{\tau \le t} = | ||
\\ | ||
\E\exp(-vX_\tau - \tau g(v))\chi_{\tau \le t} \ge [\text{$X_\tau \le 0$ в силу непр. справа}] \ge | ||
\\ | ||
\E \exp(-\tau g(v))\chi_{\tau \le t} \ge P(\tau \le t)\min_{s \in [0; t]} \exp(-sg(v)) | ||
\end{multline*} | ||
Таким образом | ||
\[ | ||
P(\tau \le t) \le e^{-v\eta_0}\max_{s \in [0; t]} \exp(sg(v)) | ||
\] | ||
Нужно происследовать выражение справа на минимум. | ||
\begin{itemize} | ||
\item $g(v) \le 0$. Тогда $s = 0$ и $\max = 1$. Из всех таких $v$ нужно брать максимальное, чтобы сомножитель $e^{-v\eta_0}$ становился как можно меньше. | ||
|
||
\item $g(v) \ge 0$. Тогда $s = t$ и $\max = e^{tg(v)} \ge 1$. Оценка получается $P(\tau \le t) \le \exp(tg(v) - v\eta_0)$. Требуется исследовать на минимум $g_1(v) = tg(v) - v\eta_0$ | ||
\end{itemize} | ||
Для начала поймём поведение $g$. Посчитаем первую и вторую производные: | ||
\[ | ||
g'(v) = \lambda\psi'(v) - c;\ \ g''(v) = \lambda\psi''(v) | ||
\] | ||
Так как $\psi(v) = \E e^{v\eta_1}$, то $\psi'(v) = \E \eta_1 e^{v\eta_1}$ и $\psi''(v) = \E \eta_1^2e^{v\eta_1}$. Значит, $g''(v) \ge 0$ при $v \ge 0$, то есть $g$ выпукла вниз. При этом $g(0) = 0$ и $g'(0) = \lambda a - c < 0$, а значит у $g$ существует единственный корень $v_0 > 0$ уравнения $g(v) = 0$. Этот корень даёт оценку | ||
\[ | ||
P(\tau \le t) \le e^{-v_0\eta_0} | ||
\] | ||
\textcolor{red}{Поиск минимума $g_1(v)$ просто не производим, либо можно как-то показать, что $v_0$ будет лучшим вариантом и для него.} | ||
\end{solution} |
Oops, something went wrong.